0% found this document useful (0 votes)
135 views32 pages

Critical Reasoning 3: Solutions: Assumptions: Q1

The passage discusses a critical reasoning question about an editorial indicating that employee retirement benefits are the fastest growing part of a city's public transportation agency's budget, which has a shortfall that could lead to service cuts and job losses. The question's reasoning is that for the editorial's conclusion - that accepting cuts to retirement benefits is in employees' best interest - to be valid, cutting retirement benefits must be sufficient to resolve the budget shortfall. Otherwise, accepting such cuts may not actually be in the employees' best interest.

Uploaded by

SriramShraff
Copyright
© © All Rights Reserved
We take content rights seriously. If you suspect this is your content, claim it here.
Available Formats
Download as PDF, TXT or read online on Scribd
0% found this document useful (0 votes)
135 views32 pages

Critical Reasoning 3: Solutions: Assumptions: Q1

The passage discusses a critical reasoning question about an editorial indicating that employee retirement benefits are the fastest growing part of a city's public transportation agency's budget, which has a shortfall that could lead to service cuts and job losses. The question's reasoning is that for the editorial's conclusion - that accepting cuts to retirement benefits is in employees' best interest - to be valid, cutting retirement benefits must be sufficient to resolve the budget shortfall. Otherwise, accepting such cuts may not actually be in the employees' best interest.

Uploaded by

SriramShraff
Copyright
© © All Rights Reserved
We take content rights seriously. If you suspect this is your content, claim it here.
Available Formats
Download as PDF, TXT or read online on Scribd
You are on page 1/ 32

 

Critical Reasoning 3: Solutions

Assumptions:

Q1:

Situation

An editorial indicates that the fastest growing part of a particular city's public
transportation agency's budget is employee retirement benefits. The agency's
budget has a shortfall. If the shortfall is not resolved, transportation service will
be cut and many of the employees will lose their jobs.

Reasoning
What must be true if we are to accept the editorial's conclusion, that it is in the
employee's best interest to accept cuts in retirement benefits? If cutting the
employees’ retirement benefits would not be sufficient to ​resolve the budget
shortfall​, then it may well not be in employees’ best interest to accept such cuts.
The reasons given for accepting the cuts are related to the undesirable
consequences of a continued shortfall.

(A) The argument is about whether the union's accepting the cuts is in fact in the
employees’ best interest, and does not require any assumption about whether
employees should accept what is in their best interest.

(B) This is incompatible with an assumption made by the argument. The argument
assumes that cutting the retirement benefits of the employees would help resolve
the budget shortfall.
 
(C) The argument would actually be stronger if it were NOT the case that it is in
the employees’ interest to have exceptionally generous retirement benefits.

(D) Correct. If cutting retirement benefits would not help resolve the agency's
budget shortfall, then cutting those benefits would not help the employees avoid
job cuts.

(E) The argument does not depend on assuming anything regarding what factors
would motivate the union's acceptance.
Q2:
 
Pre-thinking :
Modern technology leads to loss of self sufficiency.
Modern technology diminishes the overall well-being.
However, there is no correlation shown between "​self sufficiency​" and
"​well-being​".

(A) Physical labor is essential to a fulfilling life ​--> Incorrect, too extreme to be true
(B) Self-sufficiency contributes to a person’s well-being ​--> Correct, matches our
pre-thinking and the assumption required for drawing the conclusion from the
claim
(C) People are not free if they must depend on anything other than their own
capabilities ​--> Out of scope
(D) Anything causing a loss in life’s charm is unjustifiable unless this loss is
compensated by some gain ​--> Out of scope
(E) Technology inherently limits the well-being of its users ​--> Though a
contender, the option can not be an assumption, for the same has already been
stated in the argument i.e, "​much modern technology diminishes the overall
well-being of its users​"

Hence, ​B​.
Q3:
 
A. The demand for air travel will increase in the future.
-> Negate it: Demand will not increase-> if demand doesn't increase then there is
no point to launch jumbo jet (more seat)
B. Increased production expenses for the new jumbo jet will be offset by
increased revenues.
->Negate it: Expenses will not be offset-> This means company will have loss-> If
loses why they would do it!!

C. Passengers have no preference between the new double-decker jumbo jet and
the previous models in the market.
->Negate it: Passengers have preference-> Then they might not like jumbo jet ->
Then again loss for company

D. The new jumbo jet will be technologically unreliable once in operation,


resulting in unexpected costs or unrealized revenues.
Correct. Negate it : It is reliable -> Strengthen the case to build Jumbo Jet

E. The new jumbo jet will not require substantial new training of the pilots or
building new parking space at the airports
-> Negate it: Training is needed and new space is also needed-> Airports will not
like this-> Loss to company & not a great idea to build Jumbo jet
Q4:
 
Having an efficient, attractive subway system makes good economic sense. So,
the city needs to purchase new subway cars, since the city should always do what
makes good economic sense.

The conclusion drawn above follows logically if which one of the following is
assumed?

(A) The city should invest in an efficient, attractive subway system.


This simply restates the first clause of the last sentence in the passage. Out.

(B) Cost-effective subway cars are an integral part of an efficient subway system.
Cost-effective term makes this a bit extreme. Nothing about these cars being
INTEGRAL is written either.

(C) Investment in new subway cars makes better economic sense than many of
the other investment options open to the city.
Even if something else makes better economic sense - such as investing in
maintaining roads - it does not mean that the city cannot ALSO invest in subway
cars. Basically subway cars don't have to be the best option to be given an
investment greenlight, since it ALSO makes economic sense.

(D) New subway cars are financially affordable.


Wow. This one was a very close second. The turning point comes when you
consider that even IF the subway cars are not financially affordable (which I
daresay you could simply read as being COSTLY) in the broader project aspect of
the subway train system the passage states the the subway system makes
 
ECONOMIC SENSE. So even if the cars are expensive, the overall end result makes
economic sense anyway.

(E) New subway cars are required in order for the city to have a subway system
that is efficient and attractive.
The answer. Try negating this and seeing what happens. Better yet note that the
passage says that the subway cars are NEED-ed, but that this statement, when
negated, would say they are NOT REQUIRED.
Q5:
 
A. Spousal and marital difficulties were formerly responsible for many premature
returns from foreign assignments.

If helping spouses has improved expatriate retention by such a huge amount,


then unhappy spouses must have previously been responsible for lots of
premature returns. Choice (A) looks like a good answer. If unhappy spouses
contributed to employees’ leaving international assignments, helping spouses
adjust would improve the situation. Choice (B) is wrong. If spouses are already
thrilled with the international experience, their dissatisfaction is unlikely to
contribute to employees’ leaving their overseas posts. Choice (C) would support
the argument, but it’s too specific to be a necessary assumption on which the
conclusion depends (there could well be other reasons why spouses are
dissatisfied). Choice (D) doesn’t explain why helping spouses has improved
retention. Choice (E) provides an example of what companies are doing to help
spouses but isn’t the assumption that links the argument’s premises to the
conclusion. Choice (A) is the best answer.
Q6:
 
Premise:
Candidates who were leading in pre-election polls failed to win election to office.

Conclusion: American electoral system clearly possesses deep flaws and must be
discarded

Because the pre-poll leader failed to win, we are concluding that the electoral
system has deep flaws.

A. The candidate leading in pre-election polls won a majority of the popular vote
in the actual election yet still failed to be elected.

We are not assuming that the candidate won a majority of the popular vote. We
don't know what the flaws are. The flaw could be that the candidate leading in
pre-election poll did not win majority in actual election. All we know is that the
one leading in pre-election polls failed to be elected.

B. The voting population did not significantly alter its feelings on which candidate
was preferred during the time interval between the pre-election polls and the
actual election.

This is correct. We need this to be true (it is necessary) for the conclusion to hold.
We need the voting population to not significantly alter its feelings. Only then will
the pre election leader win and we won't conclude that the system has deep
flaws.

Negate it: The voting population did significantly alter its feelings.
 
If they did, then the pre-election poll leader would not win. Then the entire
argument falls apart.

C. The candidate winning the election consistently spent more money than the
candidate who was leading in pre-election polls but failed to actually win.
Irrelevant. We don't know what impact spending more money has.

D. The candidate who actually won office typically relied on a strongly negative
campaign strategy and attacked his or her opponent's personal credibility in the
final days before the election.
Again irrelevant. We don't know what impact this campaign strategy has.

E. The elections in question were for major national or state offices and received
considerable media coverage.
Irrelevant

Answer (B)
Q7:
 
The author concludes that parapsychology is a genuine scientific enterprise (GSE).
How does the author arrive at this conclusion?

● "Parapsychology is often considered a pseudoscience." - So


parapsychology is often NOT considered a GSE. Instead, it is commonly
believed to be a pseudoscience.
● Parapsychology "uses scientific methods such as controlled
experiments and statistical tests of clearly stated hypotheses to
examine the questions it raises." - Notice that the passage does not say
that parapsychology ONLY uses these two methods. 1) Controlled
experiments and 2) statistical tests of clearly hypotheses are just two
examples of scientific methods used by parapsychology.
● Based on the fact that parapsychology uses such scientific methods, the
author concludes that it is in fact a GSE.

In order for the author's logic to hold, we need to assume one of the following:

A:
The author does not claim that parapsychology can CONCLUSIVELY answer the
questions it raises. The author simply says that that parapsychology uses scientific
methods to examine the questions it raises. In other words, according to the
author, as long as it EXAMINES the questions using scientific methods, it can be
considered a GSE. Conclusively answering the questions is not necessary, so
eliminate (A).

B:
The author's only
 
requirement is that parapsychology must USE scientific
methods to examine the questions it raises. Whether these methods actually yield
results, credible or not, is irrelevant. Eliminate (B).

C:

According to the author, parapsychology is a GSE because it employs scientific


methods. But does that mean that employing scientific methods is the ONLY way
to be considered a GSE? In other words, employing scientific methods might be
sufficient, but is it necessary?

Consider the following example: Jon is an athlete because he is a basketball


player. Being a basketball player is sufficient but not necessary for being an
athlete. Just because someone is NOT a basketball player does not mean he or
she is NOT an athlete.

Similarly, employing scientific methods might not be the ONLY sufficient condition
for being a GSE. Thus, choice (C) is not a required assumption and can be
eliminated.

D:

We are given that parapsychology uses scientific methods to examine the


questions it raises. Based on that fact, the author concludes that it is a GSE. But is
employing scientific methods enough to make it a GSE? What if there are
additional criteria? In other words, in order for the author's argument to hold, the
use of scientific methods to examine questions must be a SUFFICIENT condition.
 
Choice (D) tells us that using scientific methods is a sufficient condition for being a
GSE, so this is exactly what we need. Without this assumption, we cannot safely
conclude that parapsychology is a GSE just because it uses scientific methods. (D)
looks good.

E:
We can infer that parapsychology raises questions. Are those questions "clearly
statable"? Perhaps, but we don't know for sure. Regardless, the fact that it raises
clearly statable questions does not necessarily mean that all of those questions
can actually be tested. This statement makes it sound like the questions can be
tested in controlled experiments BECAUSE parapsychology raises those questions.

But none of this is relevant to the author's argument. Given that parapsychology
raises questions and examines those questions using scientific methods, does that
make it a GSE? Choice (E) does not fill the logical gap, so eliminate this one.

(D) is the best answer.


Inferences
 
Q8:
According to the statements, the companies that own private aircraft for business
use are fully in compliance with the relevant law, which is summarized. A correct
inference will be a statement that must follow from at least part of the premises
given.
(A) It does not have to be true that the law costs the businesses money, as no
evidence about the relative costs is given.
(B) This choice is an irrelevant comparison, as the preferences of the executives
are not the concern of the statements.
(C) This choice does not have to follow, as there is no information given about the
travel arrangements made by large companies. The statements only indicate that
the majority of private planes are not owned by large companies.
(D) There is no information given about the travel arrangements of upper level
executives and no reason to believe that those with the companies discussed do
not comply with their companies’ policies.
(E) CORRECT. If, as the statements indicate, the companies are in full compliance
with this law, it must be true that the executives following their guidelines also
are.
Q9:
 
Inference questions - whether in CR or RC - are really asking you "what MUST be
true?" When you're looking for a must-be-true, you can avoid any answer choices
that are too extreme, as other posters have mentioned.

A. Individuals who eat sufficient calcium run no risk of bone depletion in old age.
"No risk" is way too extreme - we're only told that "some individuals" experience
positive change.

B. Individuals who do not undertake weight-bearing exercise have a high risk of


experiencing bone depletion late in life.
We can't claim that they have a "high risk", just that they may have a higher risk
than some individuals who do. (Note the qualifiers there)

C. Weight-lifting and intake of dietary calcium are the most effective methods of
avoiding bone depletion.
We're just told that it can be effective for some people. We have no information
comparing it to other methods, so we know nothing about "most."

D. A program of regular weight-bearing exercise and intake of dietary calcium


allows some individuals to avoid one of the deleterious effects of aging.
Note the nicely mitigating language here! It's hard to disprove "some."

E. Only weight-bearing exercise is necessary to build bone thickness, density, and


strength.
There may be other necessary factors that weren't mentioned in the argument, so
we can't claim it's the "only" one.

The answer is D.
Q10:
 
We don't have to look too deeply into this one to determine that it falls squarely
into the numbers and statistics realm. All we have is evidence, and the first piece
presents a statistical claim: The department store pays college students 20
percent less than it would pay employees from a temporary service. Next comes
the numerical claim: Add the costs of training and insurance, and the store still
pays less for college students. The correct answer must arise from the facts:
college students cost the store up to 20 percent less than do employees from a
temporary service—even after training and insurance. Prephrasing an answer
would be tough, so we should move right to testing the choices.

An 800 test taker has an intuitive sense of when to try to prephrase answers and
when to simply use a solid understanding of the given information to test the
choices. In either case, he attacks the choices aggressively.

(A) attempts to relate the amount spent on insurance for student auditors to the
total amount of their wages, but we have no basis for which to make this
comparison. The amount spent on insurance for college-student auditors can be
more, less, or equal to 20 percent of their basic wages without violating the
numerical facts presented.

(B) and (D) should have been fairly easy kills. (B) involves time, a subject not
included in the stimulus, while (D) mentions the loss of jobs at the temporary
service, even though we know nothing about the overall demand for their
auditors.

(С) is a little more subtle, but it involves how much the temporary service pays
its auditors, not how much it charges the store for them. A classic "scope shift," in
fact—and a pretty good premonition of things to come.
 
An 800 test taker always pays attention to the scope of the argument; this helps
him to axe easy and more difficult choices alike.

(E) fits, and one way we can verify that it's correct is to see what happens it it's
not true. If the cost of training college students is more than 20 percent of the
cost of hiring auditors from the temporary service, the overall cost of college
students must be higher than the cost of temporary-service auditors. That would
contradict the stimulus, so (E) must be true.
Q11:
 
The text tells us that celiac disease results when the body mistakes gluten for a
harmful pathogen, causing damage to the intestine. We are also told that gluten
is a protein found in certain grains, and that people suffering from celiac disease
must eliminate it from their diets. Finally, we are told that symptoms of the
disease include cramps, bloating, and anemia. We need to find an answer choice
that is inferable from these facts alone.

(A) Anemia is just one of several symptoms of the disease. We do not know
whether everyone who has the disease will also develop anemia.

(B) We do not know whether eliminating gluten will cure the disease, only that
people with the disease must not eat gluten. Perhaps the disease will exist
anyway in a latent form.
(C) We do not know whether the symptoms mentioned are also symptoms of
other conditions.
(D) We do not know whether gluten is found only in grains. It may exist in other
foods as well.
(E) CORRECT. If the body mistakes gluten for a harmful pathogen, then it must be
true that the body cannot always recognize harmless substances.
Q12:
 
The structure of the argument is as follows:

Premise: For many years, alcohol producers followed a self-imposed industry ban
on advertising on television.
Premise: Eventually, some producers broke the ban and began advertising their
products on television.
Premise: The producers who advertised on television generally charged less for
their products.
Conclusion: so if all producers began advertising in this fashion, overall costs to
consumers would be lower than if they did not advertise.

This stimulus is relatively easy to understand: a comparison is made between


alcohol producers who began to advertise on television and alcohol producers
who initially did not. We are told that the producers that first began to advertise
on television generally charged less for their products and that, if all advertisers
began to advertise this way (with lower prices), then the consumers’ overall costs
would be lower.

Answer choice (A): The information in the stimulus is about cost, not
consumption. We cannot assume that increased advertising will automatically
increase consumer consumption.

Answer choice (B): There is no way to know how alcohol producers who advertise
on television will behave if other producers also begin to advertise. All we know is
that if other producers begin to advertise similarly (i.e. charge less for their
product), then overall consumer costs will decrease.
 
Answer choice (C): ​This is the correct answer​. The stimulus tells us that when the
first producers broke the ban and began to advertise, they generally charged less
than non-advertising producers. So it must be true that those producers who did
not initially advertise generally charged more for their products than the
advertising producers.

Answer choice (D): The stimulus does not provide any information that would
allow us to infer how alcohol producers might have behaved without the
self-imposed advertising ban.

Answer choice (E): All that we can infer from the information provided is that if
more producers begin to advertise and subsequently lower their prices, the
overall cost to consumers will be lowered. We cannot know what effect that
would have on the price of alcohol from producers who still choose not to
advertise.
Q13:
 
The following can act as premises:

1) The municipal government finds it politically impossible either to raise fares or


to institute cost-saving reductions in service.
2) If public transportation were handled by a private firm:
i) profits would be vigorously pursued.
ii) there would be no necessity for covering operating costs with government
funds.

Clearly, one very possible assumption here is that private firms would not be
hamstrung by the same considerations as the municipal government.

(A) the private firms that would handle public transportation would have
experience in the transportation industry ​The experience, basis the given
premises, does not impact the firms' ability to pursue profits. Eliminate.
(B) political considerations would not prevent private firms from ensuring that
revenues cover operating costs ​Correct. Consistent with our thinking above.
(C) private firms would receive government funding if it were needed to cover
operating costs ​Nothing in the passage supports this - in fact, the opposite is
stated. Eliminate.
(D) the public would approve the cost-cutting actions taken by the private firm
Nothing in the passage supports this. Eliminate.
(E) the municipal government would not be resigned to accumulating merely
enough income to cover costs ​This is irrelevant because we are considering the
scenario of private firms operating the transportation, not the municipal
government. Eliminate.
Q14:
 
This is a "Draw a Conclusion" question. For such a question, you want to draw a
safe conclusion that is supported by the facts given in the argument and that
relies on no additional info or assumptions. Ask yourself "Which of the following
can I prove using the argument?"

The argument in a nutshell


Two types of migraines: Common and Classical
Common Migraine sufferers: Siblings and Spouses twice as likely to also have.
Classical migraine sufferers: Siblings four times as likely to also have. Spouses-->
no difference.

(A) CORRECT. Classical migraines affect only siblings, not spouses, as common
migraines do. Also, the numbers given support this statement (4x for classical
migraine siblings, 2x for common migraine siblings).

(B) Unmarried people are not specifically mentioned by the argument.

(C) People who do NOT suffer from migraines are not mentioned by the
argument.

(D) Children of migraine sufferers are not mentioned by the argument, only
spouses and siblings. Omission of a mention of the children does not imply that
they aren't affected by migraines!

(E) To prove this, you would have to make assumptions about not only the % of
the population affected by migraines, but also the % of the population that is
married or has a sibling.
 
Q15:
 
This is an Inference question, as demonstrated by the phrase, “which of the
following conclusions is
best supported by...” Since the question is Iooking for a conclusion that must be
true based off of the
information given, our first line of attack for thìs problem is to evaluate whether
each answer choice
contains extra information not found in the premises.

Answer choice “A” contains extra information, and thus cannot be true. Proving
that something is
“causing” something else is a high standard, and there Is nothing in the premises
that explicitly shows
the decrease in the United States crime rate caused the decrease in New York’s
crime rate. Correlation
does not mean causation.

The body of the question does not contain any information on Iaw enforcement
spending, so answer
choice “B” fails the “no new information” filter. A conclusion that contains new
information may or
may not be true.

Answer choice “C” contains a prediction of the future, implying that the Iaw
enforcement techniques
of one area could be successfully applied to another area. Such conclusions are
also very difficult to
 
prove. The Testmaker is hiding two fatal mistakes in answer choice First, it
assumes that the
techniques of a subsample (New York City) can be successfully applied to a larger
area (the United
States.) This Is an overgeneralization. Second, it predicts future effects based on
present trends. Until
the future is known, this can never be explicitly “proven. There is no information
about the future.

The Testmaker tries to hide answer choice “D” by requiring a bit of math and
hoping the novice test
taker confuses percentage changes with actual values. The violent crime rate in
the United States
steadily dropped 28%, down to 500 violent crimes per 100,000 people. If we
notice that 28% is
approximately 2/7, it is very easy to approximate what the crime rate was before
the drop:

500 = 0.72 x that is roughly 700

Therefore, the violent crime rate in the United States steadily decreased from 700
to 500 (violent
crimes per 100,000). Since the violent crime rate of New York City decreased from
1,861 to 851 (violent
crimes per 100,000) during this same time period, New York City’s rate was
consistently higher than
the national average. Answer choice D is true, and therefore is the correct
conclusion.
 
Answer choice “E” contains another prediction of the future. As mentioned
earlier, until the future is
known, this can never be explicitly uproven “ There is no information about the
future in the problem,
so “E” contains additional information. We cannot know what will soon be.
Q16:
 
First of all, we need to understand the argument. In Summary: Pravastatin is
supposed to lead to 3 different things:
1) Low Cholestrol
2) Low risk of Heart Disease
3) Lesser Deaths

Now the conclusion is mentioned as 1 leads to 2 i.e lowering cholesterol levels


reduces the risk of heart disease.
The answer can be to prove that 1 is mutually exclusive with 2 and/or 1 and 2 are
both linked to Pravastatin

Options:

(A) neglects the possibility that pravastatin may have severe side effects :
Irrelevant. Even if thisdrug has side effects, it doesn't weaken that low cholesterol
can lead to low heart disease

(B) fails to consider that pravastatin may reduce the risk of heart disease but not
as a consequence of its lowering cholesterol levels ​Correct captures the solution
mentioned above
 
(C) relies on past finding, rather than drawing its principal conclusion from the
data found in the specific study cited ​There is no discussion about past finding

(D) draws a conclusion regarding the effects of lowering cholesterol levels on


heart disease, when in fact the conclusion should focus on the relation between
pravastatin and cholesterol levels. ​This statement in entirety is true, but it doesn't
weaken the argument

(E) fails to consider that percentage of the general population might be taking
pravastatin ​However big or small population might be taking the drug, still low
cholesterol can lead to low risk of heart disease.
Q17:
 
Conclusion: If a story has been repeated by enough people then it is more likely to
be true.

Premise on which it is based (why): People would not repeat stories that they did
not believe were true.

Clearly the gap is that mere belief of people about the truth of a story does not
necessitate the story being true. This is hinted at earlier in the stimulus, where it
is stated, "...but neither are they particularly motivated to investigate deeply to
determine if the tale they are telling is true".

Let us consider the options.

(A) does not specify how many people need to repeat a story before someone is
justified believing it.

The flaw here is that people believing something to be true does not necessarily
make it true. This flaw is not addressed by this answer option. It merely adds
another factor of number of people that tend to repeat a story. The same flaw
continues to exist - whether few or a lot of people repeat a story does not give an
indication of whether the story is true or not.
 
(C) fails to address the claim that people will not verify the truth of a story that fits
their purpose.

This hits the nail on the head and addresses the flaw that we had identified in our
pre-thinking. Clearly the best answer choice among those given.
Q18:
 
We have a theory that P and M are significantly reduced at the age of 80
compared to 30. To break that theory a study says that a card game - specifically
designed to test P and M - says there was no difference, though. How do we still
counter that counter? You could say the methods to test P and M were
questionable. You could say that the younger people were distracted by
something in the room that day. You could also say (E) - that you only need a bare
minimum amount of P and M to play so it's not testing your MAX P and M or
Efficient P and M.

It is widely believed that by age 80, perception and memory are each significantly
reduced from their functioning levels at age 30. However, a recent study showed
no difference in the abilities of 80-year-olds and 30-year-olds to play a card game
devised to test perception and memory. Therefore, the belief that perception and
memory are significantly reduced by age 80 is false.

The reasoning above is most vulnerable to criticism on the grounds that it fails to
consider the possibility that

(A) the study’s card game does not test cognitive abilities other than perception
and memory
Irrelevant.
 

(B) card games are among the most difficult cognitive tasks one can attempt to
perform
Irrelevant, I daresay. But this would strengthen the argument we are seeking to
criticize.

(C) perception and memory are interrelated in ways of which we are not currently
aware
This is a 'but there could be something else afoot' answer choice. Not the best
criticism, right?

(D) the belief that 80-year-olds’ perception and memory are reduced results from
prejudice against senior citizens
Clearly out.

(E) playing the study’s card game perfectly requires fairly low levels of perception
and memory
Bingo and we're done.
 
Note: Some answers have been lifted off GMATCLUB, and so the credit to the
respective source, wherever due.

You might also like